Vous êtes sur la page 1sur 3

IrMO { Rd 1

November 2013

27th Irish Mathematical Olympiad


Solutions Round 1
Find the largest three-digit number whi h is divisible by 9 and in
whi h ea h digit is larger than the digit to its left.
Problem 1.

Solution. The largest three-digit number in whi h ea h digit is larger than the
digit to its left is 789, but this number is not divisible by 9. The next three su h
numbers are 689, 679, 678 and again none of these is divisible by 9. Therefore,
the leftmost digit an be at most 5. There are six three-digit number starting with 5 in whi h ea h digit is larger than the digit to its left. These are
589, 579, 578, 569, 568, 567. Among them only 567 is divisible by 9.
Therefore, 567 is the number we were looking for.
Remark: In this solution it was useful to know that an integer is divisible by 9 if
and only if the sum of its digits is divisible by 9.
Problem 2. When I was young I had a big box of blo ks, identi al ubes. They

ame pa ked in 10 rows of ten. A good few have been lost, but the other day my
little brother made a wall with them, with one blo k on top, a row of two under
that, a row of three under that, and so on down. Cleaning up after him, I noti ed
that he had used all the blo ks, and that taken down they ould still be arranged
in a full square. How many blo ks do we have now?
Solution. If the number of rows in the wall is k, the number of blo ks is equal
k(k + 1)
to 1 + 2 + 3 + . . . + k =
. Su h numbers are alled triangular numbers.
2

Be ause the blo ks an still be arranged in a full square, their number is equal
to m2 for some integer m < 10. So we look for possible k, m su h that m2 =
k(k + 1)
. To nd them, we list all squares and all triangular numbers below
2
100. The squares are 1, 4, 9, 16, 25, 36, 49, 64, 81 and the triangular numbers are
1, 3, 6, 10, 15, 21, 28, 36, 45, 55, 66, 78, 91. Only 1 and 36 appear in both lists. In
the question we talk about blo ks (plural), so there was more than one blo k
available. Therefore, the total number of blo ks is 36.
Problem 3. If x is a real number su h that x +

(Hint: The answer is an integer.)

1
1
= 3, nd the value of x3 + 3 .
x
x

Solution. Without al ulating x we an nd the answer. The main observation


3

3

is x + x1 = x3 + x13 + 3 x + x1 . Therefore, x3 + x13 = x + x1 3 x + x1 =
33 3 3 = 27 9 = 18.
1
An alternative solution rst solves the equation x + = 3 for x. This amounts of
x

solving the quadrati equation x2 3x + 1 = 0 whose solutions are 12 (3 5). To

IrMO { Rd 1

November 2013

2
3
2
al ulate x3 we
make
use of
x = 3x 1, whi h implies x = 3x x = 39 41 5.

Be ause
9 + 4 5 9 4 5 = 81 16 5 = 1, we now see that x + x3 =
(9 + 4 5) + (9 4 5) = 18.

The triangle ABC has a right angle at A. The point D is the


mid-point of BC and ABC = 50 . Find the measure of the angle ADC.
Problem 4.

Solution. Be ause BAC = 90 , the ir um ir le of ABC has BC as a diameter


and D is the ir um entre. Therefore, |AD| = |BD| and so BAD = ABD = 50 .
A
b

b
b

Be ause ADC is an exterior angle of the triangle BDA, the Exterior Angle Theorem now gives ADC = BAD + ABD = 50 + 50 = 100 .
In a magi square, the sum of the numbers in ea h row, olumn
and diagonal is the same. What number will repla e the question mark if the table
below is ompleted to a magi square?
Problem 5.

11
?

10

Solution. Let the numbers A, B, C, D, E, F be lled in as shown to omplete the


table to a magi square. We are asked to nd D.
6

11

10

Comparing the sum in rst row with the sum in one of the diagonals gives the
equation 6 + 11 + A = 10 + C + A, hen e 6 + 11 = 10 + C, whi h implies C = 7.
Comparing now the left olumn with the middle row, we get 6+B+10 = B+C+D,
hen e 6 + 10 = C + D = 7 + D whi h implies D = 9.
2

IrMO { Rd 1

November 2013

Problem 6. Given that the triangle ABC has ABC = 30 , what is the largest
|AB|
possible value that
an have?
|AC|
Solution. We present three di erent solutions.
First Solution: Re e t the triangle ABC in the line BC to obtain the triangle
BA C. Be ause ABA = 60 and |AB| = |A B|, the triangle ABA is equilateral.
A
b

30
30
b

C
b

A
This implies that |AB| = |AA | and |AC| = |CA |. The triangle inequality gives
|AA | |AC| + |CA | and so |AB| = |AA | |AC| + |CA | = 2|AC|. This shows that
|AB|
2 with equality if and only if C is on the line AA (i.e. ABC has a right
|AC|
|AB|
angle at C). Therefore, 2 is the largest possible value
an have.
|AC|

If your students know that sin (30 ) = 12 , you an solve this


problem with the aid of the de nition of the sine of an angle. Let D be the foot
of the altitude from A.

Se ond Solution:

30
b

C
D
Then |AD| |AC| with equality if and only if C = D, i.e. ABC has a right angle
= sin (ABC) = sin (30 ) = 21 , we see now that |AB| = 2|AD|
at C. Be ause |AD|
|AB|
|AB|
2|AC|, whi h gives
2 with equality if and only if C = D. Therefore, 2 is
|AC|
|AB|
an have.
the largest possible value
|AC|
|AB|
|AC|
If your students know the sine rule (BCA)
= (ABC)
, you an
1
argue as follows. Be ause sin (ABC) = sin (30 ) = 2 and sin (BCA) 1 with
equality if and only if BCA = 90 , the sine rule implies

Third Solution:

sin

sin

|AB|
sin (BCA)
=
= 2 sin (BCA) 2
|AC|
sin (ABC)

with equality if and only if BCA = 90 . Therefore, 2 is the largest possible value
|AB|
an have.
|AC|

Vous aimerez peut-être aussi